Fin 650 Exam 1

Lakukan tugas rumah & ujian kamu dengan baik sekarang menggunakan Quizwiz!

Assume you purchased 500 shares of XYZ common stock on margin at $105 per share fromyour broker. If the initial margin is 55%, the amount you borrowed from the broker is _______

500× $105 × 0.45 = $23,625

In calculating the Dow Jones Industrial Average, the adjustment for stock splits occurs_______

by adjusting the denominator

According to historical data, over the long run which of the following assets has the bestchance to provide the best after-inflation, after-tax rate of return?

common stocks

A bond that has no collateral is called a _______

debenture

Eurodollars are _______.

dollar-denominated deposits at any foreign bank or foreign branch of an American bank

Building a zero-investment portfolio will always involve _________

equal investments in a short and a long position

Consider an investment opportunity set formed with two securities that are perfectly negativelycorrelated. The global minimum-variance portfolio has a standard deviation that is always__________.

equal to 0

If enough investors decide to purchase stocks, they are likely to drive up stock prices, therebycausing __________ and __________

expected returns to fall; risk premiums to fall

If all investors become more risk averse, the SML will __________

have the same intercept with a steeper slope

According to Tobin's separation property, portfolio choice can be separated into two independenttasks consisting of __________ and __________.

identifying the optimal risky portfolio; constructing a complete portfolio from T-bills and the optimalrisky portfolio based on the investor's degree of risk aversion

Preferred stock is like long-term debt in that ______

it promises to pay to its holder a fixed stream of income each year

On a standard expected return versus standard deviation graph, investors will prefer portfoliosthat lie __________ the current investment opportunity set.

left and above

Some diversification benefits can be achieved by combining securities in a portfolio as long asthe correlation between the securities is __________.

less than 1

The possibility of arbitrage arises when _______

mispricing among securities creates opportunities for riskless profits

The expected return on the market portfolio is 14%. The risk-free rate is 7%. The expected returnon SDA Corporation common stock is 13%. The beta of SDA Corporation common stock is 1.60.Within the context of the capital asset pricing model, __________.

αSDA = 0.13 − [0.07 + 1.60(0.14 − 0.07)] = −0.0520 = −5.20%

An investor can design a risky portfolio based on two stocks, A and B. The standard deviation ofreturn on stock A is 20%, while the standard deviation on stock B is 15%. The correlation coefficientbetween the returns on A and B is 0%. The rate of return for stocks A and B is 20% and 10%respectively. The expected return on the minimum-variance portfolio is approximately _______

wA = σB^2/(σB^2 + σA^2) = 0.15^2/(0.15^2 + 0.20^2) = 0.36 wB = 1 − wA = 1 − 0.36 = 0.64 E(rP) = 0.20 × 0.36 + 0.10 × 0.64 = 0.136 = 13.6%

The __________ measure of returns ignores compounding

Arithmetic average is just the sum of returns in each period divided by the number of perio

Suppose you pay $9,400 for a $10,000 par Treasury bill maturing in 6 months. What is theeffective annual rate of return for this investment?

($10,000/$9,400)^(12/6) − 1 = 0.1317 = 13.17%

Asset A has an expected return of 22% and a standard deviation of 26%. The risk-free rate is 9%.What is the reward-to-variability ratio?

(0.22 − 0.09)/0.26 = 0.50

An investment earns 10% the first year, earns 15% the second year, and loses 12% thethird year. The geometric average return over the 3 years was __________

(1.10 × 1.15 × 0.88 )^(1/3)− 1 = 3.64%

You have a $48,000 portfolio consisting of Intel, GE, and Con Edison. You put $20,000 in Intel,$11,200 in GE, and the rest in Con Edison. Intel, GE, and Con Edison have betas of 1.3, 1, and 0.8,respectively. What is your portfolio beta?

(20.0/48)(1.3)+(11.2/48)(1.0)+(16.8/48)(0.8)=1.055

Which one of the following is a true statement regarding corporate bond?

A corporate convertible bond gives its holder the right to exchange it for a specified numberof the company's common shares

You decide to purchase an equal number of shares of stocks of firms to create a portfolio. Ifyou wanted to construct an index to track your portfolio performance, your best match for yourportfolio would be to construct _______

a price-weighted index

If you want to know the portfolio standard deviation for a three-stock portfolio, you will have to__________.

calculate three covariances

Consider a Treasury bill with a rate of return of 5% and the following risky securities:Security A: E(r) = 0.15; variance = 0.0400Security B: E(r) = 0.10; variance = 0.0225Security C: E(r) = 0.12; variance = 0.1000Security D: E(r) = 0.13; variance = 0.0625The investor must develop a complete portfolio by combining the risk-free asset with one of thesecurities mentioned above. The security the investor should choose as part of her completeportfolio to achieve the best CAL would be __________

security A

Initial margin requirements on stocks are set by _______

the Federal Reserve

An important characteristic of market equilibrium is __________.

the absence of arbitrage opportunities

The holding period return on a stock is equal to __________

the capital gain yield over the period plus the dividend yield

The market risk premium is best approximated by _________

the difference between the return on an index fund and the return on Treasury bills

The term complete portfolio refers to a portfolio consisting of _________

the risk-free asset combined with at least one risky asset

The reward-to-volatility ratio is given by __________

the slope of the capital allocation line

Money market securities are sometimes referred to as cash equivalents because _______

they are safe, marketable, and offer low return

You invest $1,000 in a complete portfolio. The complete portfolio is composed of a riskyasset with an expected rate of return of 16% and a standard deviation of 20% and a Treasurybill with a rate of return of 6%. __________ of your complete portfolio should be invested in therisky portfolio if you want your complete portfolio to have a standard deviation of 9%.

σC = wP × σP0. 09 = wP × 0.20 Wp=45%

The return on the risky portfolio is 15%. The risk-free rate, as well as the investor'sborrowing rate, is 10%. The standard deviation of return on the risky portfolio is 20%. If thestandard deviation on the complete portfolio is 25%, the expected return on the completeportfolio is __________.

σC = wP × σP= wP × 0.20 = 0.25 wP = 1.25 wf = 1 − 1.25 = −0.25 E(rC) = 1.25 × 0.15 − 0.25 × 0.10 = 0.1625 = 16.25%

Which one of the following is a true statement

Common dividends cannot be paid if preferred dividends are in arrears on cumulativepreferred stock

Two investment advisers are comparing performance. Adviser A averaged a 20% return with aportfolio beta of 1.5, and adviser B averaged a 15% return with a portfolio beta of 1.2. If the T-billrate was 5% and the market return during the period was 13%, which adviser was the better stockpicker?

E(r) = rf + β × [E(rM) − rf] E(rA) = 0.05 + 1.5 × [0.13 − 0.05] = 0.170 = 17.00% E(rB) = 0.05 + 1.2 × [0.13 − 0.05] = 0.146 = 14.60% α = E(r) − {rf + β × [E(rM) − rf]}αA = 0.20 − 0.17 = 0.03 = 3.00% αB = 0.15 − 0.146 = 0.004 = 0.40% Advisor A was better because he generated a larger alpha.

Consider the CAPM. The risk-free rate is 5%, and the expected return on the market is 15%.What is the beta on a stock with an expected return of 17%?

E(r) = rf + β × [E(rM) − rf]0.17 = 0.05 + β × [0.15 − 0.05]Solve for β:β = 1.2

You put up $50 at the beginning of the year for an investment. The value of the investmentgrows 4% and you earn a dividend of $3.50. Your HPR was _________

HPR = $50.00 × 1.04 + $3.50 − $50.00$50.00 = 0.11 = 11.00%

An investor puts up $5,000 but borrows an equal amount of money from his broker to doublethe amount invested to $10,000. The broker charges 7% on the loan. The stock was originallypurchased at $25 per share, and in 1 year the investor sells the stock for $28. The investor'srate of return was _______.

HPR = (($28 − $25) × 400 − 0.07 × $5,000)/$5,000= 0.17=17%

The Hydro Index is a price weighted stock index based on the 5 largest boat manufacturersin the nation. The stock prices for the five stocks are $10, $20, $80, $50 and $40. The price ofthe last stock was just split 2 for 1 and the stock price was halved from $40 to $20. What is thenew divisor for a price weighted index?

Hydro Index old ($10 + $20 + $80 + $50 + $40)/5 = $40 Hydro Index new ($10 + $20 + $80 + $50 + $20)/X = $40 Solve for X: X = 4.50

Three stocks have share prices of $12, $75, and $30 with total market values of $400million, $350 million, and $150 million, respectively. If you were to construct a price-weightedindex of the three stocks, what would be the index value?

Index = $12 + $75 + $303 = $39

You run a regression for a stock's return on a market index and find the following Excel output:Multiple R 0.35R-Square 0.12Adjusted R-Square 0.02Standard Error 38.45Observations 12CoefficientsStandardError t-Stat p-ValueIntercept 4.05 15.44 0.26 0.80Market 1.32 0.97 1.36 0.10__________ of the total risk is from systematic risk factors

R2 = 12 means 12% of the variance is explained by the regression, and the percentage of systematicrisk

Consider the following two investment alternatives: First, a risky portfolio that pays a 15%rate of return with a probability of 40% or a 5% rate of return with a probability of 60%. Second,a Treasury bill that pays 6%. The risk premium on the risky investment is ________

Risk premium = (0.4 × 0.15 + 0.6 × 0.05) − 0.06 = 0.03

You invest $1,000 in a complete portfolio. The complete portfolio is composed of a riskyasset with an expected rate of return of 16% and a standard deviation of 20% and a Treasurybill with a rate of return of 6%. The slope of the capital allocation line formed with the risky assetand the risk-free asset is approximately __________

Slope = (E(rP) - rf)/σP = 0.16 − 0.060.20 = 0.50

The figures below show plots of monthly excess returns for two stocks plotted against excessreturns for a market index.Which stock is likely to further reduce risk for an investor currently holding her portfolio in a well-diversified portfolio of common stock?

Stock A (The one that is not linear)

Asset A has an expected return of 15% and a reward-to-variability ratio of 0.4. Asset B has anexpected return of 20% and a reward-to-variability ratio of 0.3. A risk-averse investor would prefer aportfolio using the risk-free asset and __________.

The reward-to-risk ratio is the important ranking metric. The highest reward-to-risk ratio is preferred. Asset A

TIPS are _______

Treasury bonds that protect investors from inflation

You short-sell 200 shares of Tuckerton Trading Company, now selling for $50 per share.What is your maximum possible gain, ignoring transactions cost?

Tuckerton could go bankrupt, with a share price of $0. You could keep the entire proceeds fromthe short sale.Maximum gain = Proceeds − Minimum possible replacement cost= 200 × $50 − 200 × $0= $10,000

You short-sell 200 shares of Tuckerton Trading Company, now selling for $50 per share.What is your maximum possible loss

Unlimited

Which of the following statistics cannot be negative?

Variance cannot be negative as it is the square of standard deviation.

Which risk can be partially or fully diversified away as additional securities are added to aportfolio? 1. Total risk 2. Systematic risk 3. Firm-specific risk

. 1 and 3

A portfolio is composed of two stocks, A and B. Stock A has a standard deviation of return of 16%,while stock B has a standard deviation of return of 22%. Stock A comprises 60% of the portfolio,while stock B comprises 40% of the portfolio. If the variance of return on the portfolio is 0.031, thecorrelation coefficient between the returns on A and B is _______

0.031 = (0.62)(0.162) + (0.42)(0.222) + 2(0.6)(0.4)(0.16)(0.22) ρ;ρ = 0.831

A portfolio with a 25% standard deviation generated a return of 15% last year when T-billswere paying 4.5%. This portfolio had a Sharpe ratio of __________

0.42

Treasury notes have initial maturities between _______ years

1 and 10

Rank the following from highest average historical return to lowest average historical returnfrom 1926 to 2019 .1. Small stocks 2. Long-term bonds 3. Large stocks 4. T-bills

1, 3, 2, 4

Risk that can be eliminated through diversification is called __________ risk

All of these options are correct

5. What is the expected return on the market? (With a chart)

Answer will always be where Beta is 1

An investor buys $8,000 worth of a stock priced at $40 per share using 50% initial margin.The broker charges 6% on the margin loan and requires a 30% maintenance margin. In 1 yearthe investor has interest payable and gets a margin call. At the time of the margin call thestock's price must have been less than _______. (hint: interest rate is charged towards yourmargin)

At the time of purchases:Stock $ 8,000 Loan 4,000Equity 4,000Total Assets $ 8,000 TL + E 8,000After one year:Stock 200 × P Loan $ 4,000Interest payable $ 240Equity 200 × P−4,000−240Total Assets200 × P TL + E 200 × PMargin Call when Equity/Market Value < 0.30($200 × P − $4,000 − $240)/200 × P < 0.30 → P = $30.29Margin Call when stock price drops below $30.29

You sold short 300 shares of common stock at $30 per share. The initial margin is 50%. Youmust put up _________

Investment = 300 × $30 × 0.50 = $4,500

A benchmark market value index is comprised of three stocks. Yesterday the three stockswere priced at $22, $32, and $70. The number of outstanding shares for each is 780,000shares, 680,000 shares, and 380,000 shares, respectively. If the stock prices changed to $26,$30, and $72 today respectively, what is the 1-day rate of return on the index

$26 × 780,000 + $30 × 680,000 + $72 × 380,000$22 × 780,000 + $32 × 680,000 + $70 × 380,000]−1=3.85%

Assume you purchased 500 shares of XYZ common stock on margin at $40 per share fromyour broker. If the initial margin is 60%, the amount you borrowed from the broker is ______

500 × $40 × 0.40 = $8,000

You sell short 700 shares of Microsoft that are currently selling at $60 per share. You postthe 50% margin required on the short sale. If you earn no interest on the funds in your marginaccount, what will be your rate of return after 1 year if Microsoft is selling at $54? (Ignore anydividends.)

$60 − $54/0.5 × $60 = 20%

You sell short 300 shares of Microsoft that are currently selling at $30 per share. You postthe 50% margin required on the short sale. If you earn no interest on the funds in your marginaccount, what will be your rate of return after 1 year if Microsoft is selling at $27? (Ignore anydividends.)

($30 − $27)/(0.5 × $30) = 20%

You purchased 250 shares of common stock on margin for $25 per share. The initial marginis 65%, and the stock pays no dividend. Your rate of return would be _______ if you sell thestock at $32 per share. Ignore interest on margin

($32 − $25)/($25 × 0.65)=0.43 43%

A stock quote indicates a stock price of $70 and a dividend yield of 2%. The latest quarterlydividend received by stock investors must have been ______ per share

($70 × 0.020)/4 = $0.35

Consider the single factor APT. Portfolio A has a beta of 1.3 and an expected return of 23%.Portfolio B has a beta of 0.8 and an expected return of 19%. The risk-free rate of return is 8%. If youwanted to take advantage of an arbitrage opportunity, you should take a short position in portfolio__________ and a long position in portfolio __________.

(0.23 − 0.08)/1.3 A < (0.19 − 0.08)/0.8 B ; Short A; Buy BShort A; Buy B, Beta A - 1.30, Beta B - 0.8

5. Based on the outcomes in the following table, choose which of the statements below is (are)correct?Scenario Security A Security B Security CRecession Return > E(r) Return = E(r) Return < E(r)Normal Return = E(r) Return = E(r) Return = E(r)Boom Return < E(r) Return = E(r) Return > E(r)1. The covariance of security A and security B is zero.2. The correlation coefficient between securities A and C is negative.3. The correlation coefficient between securities B and C is positive

1 and 2 only

The market portfolio has a beta of __________

1.0

A loan for a new car costs the borrower 0.8% per month. What is the EAR?

1.00812 − 1 = 0.1003 = 10.03%

Most studies indicate that investors' risk aversion is in the range _________

1.5 - 4.0

Consider the multifactor APT with two factors. Portfolio A has a beta of 0.5 on factor 1 and a betaof 1.25 on factor 2. The risk premiums on the factor 1 and 2 portfolios are 1% and 7%, respectively.The risk-free rate of return is 7%. The expected return on portfolio A is __________ if no arbitrageopportunities exist.

16.25% E(rA) = rf + β1 × R1 + β2 × R2E(rA) = 0.07 + 0.5 × 0.01 + 1.25 × 0.07 = 0.1625 = 16.25%

Security A has a higher standard deviation of returns than security B. We would expect that: 1. Security A would have a risk premium equal to security B. 2. The likely range of returns for security A in any given year would be higher than the likelyrange of returns for security B. 3. The Sharpe ratio of A will be higher than the Sharpe ratio of B

2 only

The optimal risky portfolio can be identified by finding: 1. The minimum-variance point on the efficient frontier 2. The maximum-return point on the efficient frontier and the minimum-variance point on the efficientfrontier 3. The tangency point of the capital market line and the efficient frontier 4. The line with the steepest slope that connects the risk-free rate to the efficient frontier

3 and 4 only

Treasury bills are paying a 4% rate of return. A risk-averse investor with a risk aversionof A = 3 should invest entirely in a risky portfolio with a standard deviation of 24% only if therisky portfolio's expected return, E(rQ) is at least __________

A = (E(rQ) - rf)/σ2Q 3 = (E(rQ) − 0.04)/0.24^2 = Solve for E(rQ): E(rQ) = 0.2128 = 21.28%

Which of the following is not a true statement regarding municipal bonds

A municipal bond is a debt obligation issued by the federal government

If the simple CAPM is valid and all portfolios are priced correctly, which of the situations below ispossible? Consider each situation independently, and assume the risk-free rate is 5%. A)PortfolioExpectedReturn BetaA 13% 1.1Market 13% 1.0 B)PortfolioExpectedReturnStandardDeviationA 16% 11%Market 11% 19% C)PortfolioExpectedReturn BetaA 16% 1.1Market 11% 1.0 D)PortfolioExpectedReturn BetaA 21.8% 2.1Market 13% 1.0

A) Not possible—two portfolios with different betas cannot have the same expected return B) Not possible—under CAPM market portfolio must yield highest CAL. C) Not possible—portfolio A and the market have different excess returns per unit of risk. D) Possible(22 − 5)/2.1 = (13 − 5)/1.0

Consider the single factor APT. Portfolio A has a beta of 1.3 and an expected return of 21%.Portfolio B has a beta of 0.7 and an expected return of 17%. The risk-free rate of return is 8%. If youwanted to take advantage of an arbitrage opportunity, you should take a short position in portfolio__________ and a long position in portfolio __________

A; B Compare Reward-to-Market-Risk ratios:(E(r) - rf)/β(0.21 − 0.08)/1.3<(0.17 − 0.08)/0.7→(0.21 - 0.08)/1.3<(0.17 - 0.08)/0.7→Portfolio A is overvalued relative to Portfolio B.Therefore, short Portfolio A and buy Portfolio B

You sell short 200 shares of Doggie Treats Incorporated that are currently selling at $25 pershare. You post the 50% margin required on the short sale. If your broker requires a 30%maintenance margin, at what stock price will you get a margin call? (You earn no interest on thefunds in your margin account, and the firm does not pay any dividends.

Account at the time of the short sale: Cash from SS $ 5,000.00 Liability $ 5,000.00 Cash for Equity $ 2,500.00 Equity $ 2,500.00 Total Assets $ 7,500.00 TL+E $ 7,500.00 Account as prices change: Cash from SS $ 5,000.00 Liability $ 200P Cash for Equity $ 2,500.00 Equity $ 7,500−200PTotal Assets $ 7,500.00 TL+E $ 7,500.00Margin Call when Equity/Market Value < 0.30($7,500 − 200 × P)/(200 × P) < 0.30 → P = $28.85Margin Call when stock price drops below $28.85

Two assets have the following expected returns and standard deviations when the risk-freerate is 5%:Asset A E(rA) = 10%; σA = 20%Asset B E(rB) = 15%; σB = 27%An investor with a risk aversion of A = 3 would find that __________ on a risk-return bas

Based on the relationship:A = (E(rQ) - rf)/(σQ)^2The minimum acceptable return is given by:E(rQ) = A × (σQ)^2 + rfE(rA) = 3 × 0.22 + 0.05 = 0.17 = 17.00% > 10.00%E(rB) = 3 × 0.272 + 0.05 = 0.2687 = 26.87% > 15.00%Neither portfolio generates enough return given the investor's risk aversion. neither asset A nor asset B is acceptable

An investor can design a risky portfolio based on two stocks, A and B. The standard deviation ofreturn on stock A is 24%, while the standard deviation on stock B is 14%. The correlation coefficientbetween the returns on A and B is 0.35. The expected return on stock A is 25%, while on stock B itis 11%. The proportion of the minimum-variance portfolio that would be invested in stock B isapproximately __________.

Cov(rA,rB) = ρABσAσB = 0.35 × 0.24 × 0.14 = 0.01176 WB = (σA^2 − Cov(rA,rB))/(σ2B + σ2A − 2Cov(rA,rB)) = (0.24^2 − 0.01176)/(0.14^2 + 0.24^2 − 2 ×0.01176 = 0.8539) = 85.39%

You are considering investing $1,000 in a complete portfolio. The complete portfolio iscomposed of Treasury bills that pay 5% and a risky portfolio, P, constructed with two riskysecurities, X and Y. The optimal weights of X and Y in P are 60% and 40%, respectively. X hasan expected rate of return of 14%, and Y has an expected rate of return of 10%. The dollarvalues of your positions in X, Y, and Treasury bills would be __________, __________, and__________, respectively, if you decide to hold a

E(rC) = [E(rX) × wX + E(rY) × wY] × wP + rf × wf 0.08 = [0.14 × 0.60 + 0.10 × 0.40] × (1 − wf) + 0.05 × wf Solve for wf : 0.08 = 0.124 × (1 − wf) + 0.05 × wf 0.074 × wf = 0.044 wf = 0.5946 = 59.46% Dollars values: T-Bills: 0.5946 × $1,000 ≈ $595 → Security X: (1 − 0.5946) × (0.60) × $1,000 ≈ $243 Security Y: (1 − 0.5946) × (0.40) × $1,000 ≈ $162

Your investment has a 40% chance of earning a 15% rate of return, a 50% chance ofearning a 10% rate of return, and a 10% chance of losing 3%. What is the standard deviation ofthis investment?

E(rP) = 0.4 × 0.15 + 0.5 × 0.10 + 0.10 × (−3%) = 0.107 = 10.7% σ(rP) = 0.4 × (0.15 − 0.107)2 + 0.5 × (0.10 − 0.107)2 + 0.10 × (−0.03 − 0.107) 2σ(rP) = 0.0514 = 5.14%

Research has identified two systematic factors that affect U.S. stock returns. The factors aregrowth in industrial production and changes in long-term interest rates. Industrial production growthis expected to be 3%, and long-term interest rates are expected to increase by 1%. You areanalyzing a stock that has a beta of 1.2 on the industrial production factor and 0.5 on the interestrate factor. It currently has an expected return of 12%. However, if industrial production actuallygrows 5% and int

E(rnew) = E(rold) + βIP × ΔIP + βInt RatesΔIR E(rnew) = 0.12 + 1.2 × (0.05 − 0.03) + 0.5 × (−0.02 − 0.01) = 0.129 =12.90%

You are considering investing $1,000 in a complete portfolio. The complete portfolio iscomposed of Treasury bills that pay 5% and a risky portfolio, P, constructed with two riskysecurities, X and Y. The optimal weights of X and Y in P are 60% and 40% respectively. X hasan expected rate of return of 14%, and Y has an expected rate of return of 10%. To form acomplete portfolio with an expected rate of return of 8%, you should invest approximately__________ in the risky portfolio. This will mean yo

E(rp) = 0.6 × 0.14 + 0.4 × 0.10 = 0.124 = 12.4% 0.08 = wp × 0.124 + (1 − wP) × 0.05 wP = 0.4054 = 40.54% ≈ 40% wX in complete portfolio = 0.4054 × 0.60 = 24.32% ≈ 24% wY in complete portfolio = 0.4054 × 0.40 = 16.22% ≈ 16%

Security X has an expected rate of return of 13% and a beta of 1.15. The risk-free rate is 5%,and the market expected rate of return is 15%. According to the capital asset pricing model, securityX is __________.

E(rx) = rf + βx × [E(rM) − rf]E(rx) = 0.05 + 1.15 × [0.15 − 0.05] = 0.165 = 16.50%Price and return are inversely related implying security X is overpriced.

You consider buying a share of stock at a price of $12. The stock is expected to pay a dividendof $1.60 next year, and your advisory service tells you that you can expect to sell the stock in 1 yearfor $14. The stock's beta is 1.2, rf is 15%, and E[rm] = 25%. What is the stock's abnormal return?

E[r]=[($14 − $12 + $1.60]/$12](100%) = 30%Required return = 15% + 1.2(25% − 15%) = 27%

You purchased 200 shares of ABC common stock on margin at $50 per share. Assume theinitial margin is 50% and the maintenance margin is 30%. You will get a margin call if the stockdrops below _______. (Assume the stock pays no dividends, and ignore interest on the marginloan.)

Equity = 200 × P − $5,000 Margin =( 200 × P − $5,000)/200 × P = 0.30 Solve for P :P = $35.71

The most marketable money market securities are ______

Treasury bills

Three stocks have share prices of $27, $90, and $60 with total market values of $490 million,$440 million, and $240 million, respectively. If you were to construct a price-weighted index ofthe three stocks, what would be the index value

Index = ($27 + $90 + $60)/3 = $59

A benchmark index has three stocks priced at $36, $59, and $69. The number of outstandingshares for each is 415,000 shares, 535,000 shares, and 683,000 shares, respectively. If themarket value-weighted index was 980 yesterday and the prices changed to $36, $56, and $72today, what is the new index value

Index = ($36 × 415,000 + $56 × 535,000 + $72 × 683,000)/($36 × 415,000 + $59 × 535,000 +$69 × 683,000) × 980 = 985

The margin requirement on a stock purchase is 25%. You fully use the margin allowed topurchase 100 shares of MSFT at $25. If the price drops to $22, what is your percentage loss?(no interest charged on the broker's loan)

Loss = ($22 − $25) × 100 = −$300Invested Capital = 0.25 × $25 × 100 = $625Return =−$300/$625=−0.48=−48%

If you require a real growth in the purchasing power of your investment of 8%, and youexpect the rate of inflation over the next year to be 3%, what is the lowest nominal return thatyou would be satisfied with?

Nominal rate = 1.08 × 1.03 − 1 = 11.24%

The price of a stock is $55 at the beginning of the year and $50 at the end of the year. If thestock paid a $3 dividend and inflation was 3%, what is the real holding-period return for theyear?

Nominal return on stock: (P1 − P0 + Div1)/$55 =($50 − $55 + $3)/$55 = −0.0364 = −3.64% Real return: r =( 1 + R)/(i −1) = (1 − 0.0364)/(1 + 0.03) − 1 = −0.0644 = −6.44%

A benchmark market value index is comprised of three stocks. Yesterday the three stockswere priced at $12, $20, and $60. The number of outstanding shares for each is 600,000shares, 500,000 shares, and 200,000 shares, respectively. If the stock prices changed to $16,$18, and $62 today respectively, what is the 1-day rate of return on the index

R(Index)=$16 × 600,000 + $18 × 500,000 + $62 × 200,000)/($12 × 600,000 + $20 × 500,000 +$60 × 200,000) −1 = 0.0616 = 6.16%

An investor buys $16,000 worth of a stock priced at $20 per share using 60% initial margin.The broker charges 8% on the margin loan and requires a 35% maintenance margin. The stockpays a $.50-per-share dividend in 1 year, and then the stock is sold at $23 per share. What wasthe investor's rate of return? (hint: dividend received is considered as increase in stock price

Value of stock in 1 year: $ 18,400Dividends received $ 400Interest due $ (512)Loan payoff $ (6,400)Ending account balance $ 11,888Return:$11,888/$9,600−1=23.83%

An investor can design a risky portfolio based on two stocks, A and B. Stock A has an expectedreturn of 19% and a standard deviation of return of 14.4%. Stock B has an expected return of 15%and a standard deviation of return of 4%. The correlation coefficient between the returns of A and Bis 0.50. The risk-free rate of return is 10%. The proportion of the optimal risky portfolio that should beinvested in stock A is __________.

WA = ((0.19 − 0.10)(0.04)2 − (0.15 − 0.10)(0.04)(0.144)(0.50))/((0.19 −0.10)(0.50)2 + (0.15 − 0.10)(0.144)2 − (0.19 − 0.10 + 0.15 −0.10)(0.04)(0.14)(0.50))WA = 0Since the numerator equals zero, WA = 0 without any further calculations.

Currently, the Dow Jones Industrial Average is computed by ______

adding the prices of 30 large "blue-chip" stocks and dividing by a divisor adjusted for stocksplits and large stock dividends

Which of the following is a correct expression concerning the formula for the standard deviationof returns of a two-asset portfolio where the correlation coefficient is positive?

o2p >

According to the capital asset pricing model, a fairly priced security will plot __________.

on the security market line

According to the capital asset pricing model, a security with a _________

positive alpha is considered underpriced


Set pelajaran terkait

13. Test (Unit 2) AZ STRATEGIC READING - 96.3%

View Set

Intermediate Accounting Chapter 15

View Set

Labor and delivery order questions

View Set

FIN202 Topic 8: Pricing derivatives

View Set

comm ch 8 (communication in intimate relationships)

View Set

Chapter 11 Review & Lab Questions

View Set

Psych of Personality Ch 18: Stress, Coping, Adjustment, and Health

View Set